Mathcenter Forum  

Go Back   Mathcenter Forum > คณิตศาสตร์โอลิมปิก และอุดมศึกษา > อสมการ
สมัครสมาชิก คู่มือการใช้ รายชื่อสมาชิก ปฏิทิน ข้อความวันนี้

ตั้งหัวข้อใหม่ Reply
 
เครื่องมือของหัวข้อ ค้นหาในหัวข้อนี้
  #31  
Old 29 เมษายน 2010, 14:46
Ne[S]zA's Avatar
Ne[S]zA Ne[S]zA ไม่อยู่ในระบบ
ลมปราณไร้สภาพ
 
วันที่สมัครสมาชิก: 13 กรกฎาคม 2008
ข้อความ: 1,221
Ne[S]zA is on a distinguished road
Default

อ้างอิง:
ข้อความเดิมเขียนโดยคุณ กิตติ View Post
ข้อนี้ผมขอให้AM-HM
$3+a+b+c \rightarrow (a+1)+(b+1)+(c+1)$
$AM=\dfrac{(a+1)+(b+1)+(c+1)}{3} $
$GM=\dfrac{3}{\dfrac{1}{a+1}+\dfrac{1}{b+1} +\dfrac{1}{c+1} } $
HM. นะครับ
__________________
||!<<<<iNesZaii>>>>!||
ตอบพร้อมอ้างอิงข้อความนี้
  #32  
Old 29 เมษายน 2010, 14:48
กิตติ's Avatar
กิตติ กิตติ ไม่อยู่ในระบบ
กระบี่ธรรมชาติ
 
วันที่สมัครสมาชิก: 08 พฤศจิกายน 2009
ข้อความ: 2,723
กิตติ is on a distinguished road
Default

ขอบคุณครับ...แก้แล้วครับ
__________________
"ถ้าเราล้มบ่อยๆ ในที่สุดเราจะรู้ว่าถ้าจะล้ม ล้มท่าไหนจะเจ็บน้อยที่สุด และรู้อีกว่าต่อไปทำยังไงจะไม่ให้ล้มอีก
ดังนั้นจงอย่ากลัวที่จะล้ม
"...อาจารย์อำนวย ขนันไทย
ครั้งแรกในชีวิตที่สอบคณิตสมาคมคณิตศาสตร์เมื่อปี2533...ผมได้แค่24คะแนน(จากร้อยคะแนน)
ตอบพร้อมอ้างอิงข้อความนี้
  #33  
Old 29 เมษายน 2010, 15:23
Ne[S]zA's Avatar
Ne[S]zA Ne[S]zA ไม่อยู่ในระบบ
ลมปราณไร้สภาพ
 
วันที่สมัครสมาชิก: 13 กรกฎาคม 2008
ข้อความ: 1,221
Ne[S]zA is on a distinguished road
Default

อ้างอิง:
ข้อความเดิมเขียนโดยคุณ tatari/nightmare View Post
10.(IMO 1995)ให้ $a,b,c>0$ ซึ่ง $abc=1$ จงแสดงว่า
$$\frac{1}{a^3(b+c)}+\frac{1}{b^3(c+a)}+\frac{1}{c^3(a+b)}\geq \frac{3}{2}$$
หมายเหตุ:สามารถแก้ได้โดยใช้ AM-GM อย่างเดียวได้
ขอใช้ Cauchy นะครับ
จาก $abc=1$ จะทำให้ได้ว่า $\dfrac{1}{a}+\dfrac{1}{b}+\dfrac{1}{c}=ab+bc+ca$
โดย Cauchy-Schwarz Inequality จะได้ว่า
$$\dfrac{1}{a}+\dfrac{1}{b}+\dfrac{1}{c}=\sum_{cyc} \dfrac{1}{a\sqrt{a}\sqrt{b+c}}\cdot \sqrt{a}\sqrt{b+c}\leqslant \sqrt{\sum_{cyc} \dfrac{1}{a^3(b+c)}}\cdot \sqrt{2(ab+bc+ca)}= \sqrt{\sum_{cyc} \dfrac{1}{a^3(b+c)}}\cdot \sqrt{2\Big(\dfrac{1}{a}+\dfrac{1}{b}+\dfrac{1}{c} \Big)}$$
ยกกำลังสองทั้งสองข้างจะได้
$$\dfrac{\dfrac{1}{a}+\dfrac{1}{b}+\dfrac{1}{c}}{2}=\dfrac{ab+bc+ca}{2}\leqslant \sum_{cyc} \dfrac{1}{a^3(b+c)}\_\_\_\_\_\_(*)$$
จาก $abc=1$
โดย AM.-GM. Inequality จะได้ว่า
$$\dfrac{ab+bc+ca}{3}\geqslant \sqrt[3]{\dfrac{1}{abc}}=1 ----->> ab+bc+ca\geqslant 3\_\_\_\_\_(**)$$
(**) แทนใน (*) จะได้ว่า
$$\sum_{cyc} \dfrac{1}{a^3(b+c)}\geqslant \dfrac{3}{2}$$
__________________
||!<<<<iNesZaii>>>>!||

29 เมษายน 2010 16:02 : ข้อความนี้ถูกแก้ไขแล้ว 2 ครั้ง, ครั้งล่าสุดโดยคุณ Ne[S]zA
ตอบพร้อมอ้างอิงข้อความนี้
  #34  
Old 29 เมษายน 2010, 19:44
กิตติ's Avatar
กิตติ กิตติ ไม่อยู่ในระบบ
กระบี่ธรรมชาติ
 
วันที่สมัครสมาชิก: 08 พฤศจิกายน 2009
ข้อความ: 2,723
กิตติ is on a distinguished road
Default

อ้างอิง:
ข้อความเดิมเขียนโดยคุณ หยินหยาง View Post
7. กำหนดให้ $x, y, z $เป็นจำนวนจริงจงแสดงว่า $x^4+y^4+z^4\geq xyz(x+y+z)$
$
ข้อนี้ทำแบบนี้ได้ไหมครับ
$x^4+y^4 \geqslant 2x^2y^2$
$y^4+z^4 \geqslant 2y^2z^2$
$x^4+z^4 \geqslant 2x^2z^2$
จับมาบวกกันทั้งสามอสมการได้
$x^4+y^4+z^4\geq (x^2y^2+y^2z^2+x^2z^2)$
$x^4+y^4+z^4\geq xyz(\frac{xy}{z} +\frac{yz}{x} +\frac{xz}{y} )$
จัดหน้าใหม่เป็น
$\frac{x^4+y^4+z^4}{xyz} \geq (\frac{xy}{z} +\frac{yz}{x} +\frac{xz}{y} )$
ไม่รู้ว่าการคูณไขว้ลงมานั้นจะต้องเผื่อกรณีที่$xyz<0$ซึ่งเครื่องหมายจะเปลี่ยน แต่โจทย์กำหนดว่า$x, y, z $เป็นจำนวนจริง....ถ้าผมจะผิดก็คงตรงนี้แหละครับ
มาดูเฉพาะ$\frac{xy}{z} +\frac{yz}{x} +\frac{xz}{y} $
$\frac{xy}{z} +\frac{yz}{x} \geqslant 2y$
$\frac{yz}{x} +\frac{xz}{y} \geqslant 2z$
$\frac{xy}{z} +\frac{xz}{y} \geqslant 2x$
จับสามอสมการมาบวกกันจะได้
$\frac{xy}{z} +\frac{yz}{x} +\frac{xz}{y} \geqslant x+y+z$
จาก$a>b$แล้ว$b>c$จะได้ว่า$a>c$
$\frac{x^4+y^4+z^4}{xyz} \geq (\frac{xy}{z} +\frac{yz}{x} +\frac{xz}{y} )$ และ$\frac{xy}{z} +\frac{yz}{x} +\frac{xz}{y} \geqslant (x+y+z)$
จะสรุปว่า$\frac{x^4+y^4+z^4}{xyz} \geq (x+y+z) $
ลองช่วยผมดูหน่อยเถอะครับว่า จะทำผิดขั้นตอนหรือเปล่าครับ
__________________
"ถ้าเราล้มบ่อยๆ ในที่สุดเราจะรู้ว่าถ้าจะล้ม ล้มท่าไหนจะเจ็บน้อยที่สุด และรู้อีกว่าต่อไปทำยังไงจะไม่ให้ล้มอีก
ดังนั้นจงอย่ากลัวที่จะล้ม
"...อาจารย์อำนวย ขนันไทย
ครั้งแรกในชีวิตที่สอบคณิตสมาคมคณิตศาสตร์เมื่อปี2533...ผมได้แค่24คะแนน(จากร้อยคะแนน)
ตอบพร้อมอ้างอิงข้อความนี้
  #35  
Old 29 เมษายน 2010, 19:54
Ne[S]zA's Avatar
Ne[S]zA Ne[S]zA ไม่อยู่ในระบบ
ลมปราณไร้สภาพ
 
วันที่สมัครสมาชิก: 13 กรกฎาคม 2008
ข้อความ: 1,221
Ne[S]zA is on a distinguished road
Default

ผมว่า ถ้ามี $x,y,z$ เป็น ศูนย์ อย่างน้อยหนึ่งตัวก็ิผิดละครับ
__________________
||!<<<<iNesZaii>>>>!||
ตอบพร้อมอ้างอิงข้อความนี้
  #36  
Old 29 เมษายน 2010, 20:02
กิตติ's Avatar
กิตติ กิตติ ไม่อยู่ในระบบ
กระบี่ธรรมชาติ
 
วันที่สมัครสมาชิก: 08 พฤศจิกายน 2009
ข้อความ: 2,723
กิตติ is on a distinguished road
Default

จากโจทย์ที่คุณหยินหยางกำหนดมานั้น$x,y,z$เป็นจำนวนจริง และที่ผมได้ตามดูในวิธีการเฉลยของสองท่าน จะกำหนดให้$x,y,z >0$ ซึ่งก็จริงอย่างที่คุณเนสว่า มันไม่จริงเมื่อตัวใดตัวหนึ่งเป็นศูนย์....คงต้องหาวิธีทำใหม่อีกแล้ว

มาลองแตกต่อจาก$x^4+y^4+z^4 \geqslant (xy)^2+(yz)^2+(xz)^2$
$(xy)^2+(yz)^2 \geqslant 2xy^2z$
$(yz)^2+(xz)^2 \geqslant 2xyz^2$
$(xy)^2+(xz)^2 \geqslant 2x^2yz$
จับมารวมกัน
$(xy)^2+(yz)^2+(xz)^2 \geqslant xyz(x+y+z)$
จะได้$x^4+y^4+z^4 \geqslant xyz(x+y+z)$

ลองกลับไปดูการกำหนดของการใช้$AM-GM$พบว่าในการใช้$AM-GM$ นิยามกำหนดให้
$a_1,a_2,...,a_n >0$แล้ว
$\frac{a_1+a_2+...+a_n}{n} \geqslant \sqrt[n]{a_1\times a_2\times ...\times a_n} $
ดังนั้นเราจึงใช้$AM-GM$ไม่ได้...ใช่ไหมครับ
__________________
"ถ้าเราล้มบ่อยๆ ในที่สุดเราจะรู้ว่าถ้าจะล้ม ล้มท่าไหนจะเจ็บน้อยที่สุด และรู้อีกว่าต่อไปทำยังไงจะไม่ให้ล้มอีก
ดังนั้นจงอย่ากลัวที่จะล้ม
"...อาจารย์อำนวย ขนันไทย
ครั้งแรกในชีวิตที่สอบคณิตสมาคมคณิตศาสตร์เมื่อปี2533...ผมได้แค่24คะแนน(จากร้อยคะแนน)

30 เมษายน 2010 11:41 : ข้อความนี้ถูกแก้ไขแล้ว 2 ครั้ง, ครั้งล่าสุดโดยคุณ กิตติ
เหตุผล: double post
ตอบพร้อมอ้างอิงข้อความนี้
  #37  
Old 30 เมษายน 2010, 15:20
Ne[S]zA's Avatar
Ne[S]zA Ne[S]zA ไม่อยู่ในระบบ
ลมปราณไร้สภาพ
 
วันที่สมัครสมาชิก: 13 กรกฎาคม 2008
ข้อความ: 1,221
Ne[S]zA is on a distinguished road
Default

$a_i \geqslant 0$ ไม่ใช่หรอครับ ถ้า $a_i >0$ คงจะมองรวมถึง HM. อ่ะครับ (หรือผมจำผิด)
แล้วก็ในที่นี้เราใช้ $a_1=(xy)^2$ และ $a_2=(yz)^2$ ซึ่งมากกว่าหรือเท่ากับศูนย์อยู่แล้วครับ
แต่ที่จริง จากความจริง เราได้อยู่แล้วว่า $(xy-yz)^2\geqslant 0$ ครับ
__________________
||!<<<<iNesZaii>>>>!||
ตอบพร้อมอ้างอิงข้อความนี้
  #38  
Old 30 เมษายน 2010, 17:39
กิตติ's Avatar
กิตติ กิตติ ไม่อยู่ในระบบ
กระบี่ธรรมชาติ
 
วันที่สมัครสมาชิก: 08 พฤศจิกายน 2009
ข้อความ: 2,723
กิตติ is on a distinguished road
Default

คืออย่างนี้ครับ....โจทย์ข้อนี้เขียนไว้อย่างนี้ว่า
อ้างอิง:
ข้อความเดิมเขียนโดยคุณ หยินหยาง View Post
7. กำหนดให้ $x, y, z $เป็นจำนวนจริงจงแสดงว่า $x^4+y^4+z^4\geq xyz(x+y+z)$
แต่ในนิยามที่เราใช้นั้น จะเขียนว่าเป็นจำนวนเต็มบวก(positive real number) และบางบทความเรื่องอสมการ เขาก็เขียนในส่วนของ$AM-GM$ ว่า$a_1,a_2,...,a_n \geqslant 0$
เมื่อเรานำ$AM-GM$มาใช้พิสูจน์อสมการที่ระบุว่า ตัวแปรนี้เป็นจำนวนจริงใดๆ ตามโจทย์ของคุณหยินหยางแล้ว จะใช้ได้ไหม ใช้ได้แต่เฉพาะกรณีที่$a_1,a_2,...,a_n \geqslant 0$อย่างเดียว หรือรวมไปทั้งกรณีที่ตัวแปรเป็นจำนวนจริง(ซึ่งรวมจำนวนเต็มลบด้วย)...ผมงงตรงนี้คำว่าบทนิยามจำกัดของเขตไว้น้อยกว่าขอบเขตที่โจทย์กำหน ดให้กับตัวแปร....ช่วยอธิบายให้คนแก่ขี้สงสัยฟังหน่อยเถอะครับ
__________________
"ถ้าเราล้มบ่อยๆ ในที่สุดเราจะรู้ว่าถ้าจะล้ม ล้มท่าไหนจะเจ็บน้อยที่สุด และรู้อีกว่าต่อไปทำยังไงจะไม่ให้ล้มอีก
ดังนั้นจงอย่ากลัวที่จะล้ม
"...อาจารย์อำนวย ขนันไทย
ครั้งแรกในชีวิตที่สอบคณิตสมาคมคณิตศาสตร์เมื่อปี2533...ผมได้แค่24คะแนน(จากร้อยคะแนน)

30 เมษายน 2010 17:41 : ข้อความนี้ถูกแก้ไขแล้ว 2 ครั้ง, ครั้งล่าสุดโดยคุณ กิตติ
ตอบพร้อมอ้างอิงข้อความนี้
  #39  
Old 30 เมษายน 2010, 17:43
Scylla_Shadow's Avatar
Scylla_Shadow Scylla_Shadow ไม่อยู่ในระบบ
ลมปราณไร้สภาพ
 
วันที่สมัครสมาชิก: 10 กุมภาพันธ์ 2009
ข้อความ: 1,151
Scylla_Shadow is on a distinguished road
Default

อ้างอิง:
ข้อความเดิมเขียนโดยคุณ กิตติ View Post
คืออย่างนี้ครับ....โจทย์ข้อนี้เขียนไว้อย่างนี้ว่า

แต่ในนิยามที่เราใช้นั้น จะเขียนว่าเป็นจำนวนเต็มบวก(positive real number) และบางบทความเรื่องอสมการ เขาก็เขียนในส่วนของ$AM-GM$ ว่า$a_1,a_2,...,a_n \geqslant 0$
เมื่อเรานำ$AM-GM$มาใช้พิสูจน์อสมการที่ระบุว่า ตัวแปรนี้เป็นจำนวนจริงใดๆ ตามโจทย์ของคุณหยินหยางแล้ว จะใช้ได้ไหม ใช้ได้แต่เฉพาะกรณีที่$a_1,a_2,...,a_n \geqslant 0$อย่างเดียว หรือรวมไปทั้งกรณีที่ตัวแปรเป็นจำนวนจริง(ซึ่งรวมจำนวนเต็มลบด้วย)...ผมงงตรงนี้คำว่าบทนิยามจำกัดของเขตไว้น้อยกว่าขอบเขตที่โจทย์กำหน ดให้กับตัวแปร....ช่วยอธิบายให้คนแก่ขี้สงสัยฟังหน่อยเถอะครับ
ถ้าเป็นจำนวนจริง กล่าวคือรวมจำนวนลบด้วย จะใช้ AM-GM-HM ไม่ได้เลยครับ

แต่ถ้าเป็นจำนวนจริงบวกถึงจะใช้ได้ครับ
ตอบพร้อมอ้างอิงข้อความนี้
  #40  
Old 30 เมษายน 2010, 17:52
Ne[S]zA's Avatar
Ne[S]zA Ne[S]zA ไม่อยู่ในระบบ
ลมปราณไร้สภาพ
 
วันที่สมัครสมาชิก: 13 กรกฎาคม 2008
ข้อความ: 1,221
Ne[S]zA is on a distinguished road
Default

เนื่องจาก $a_1,a_2,a_3,...,a_n\geqslant 0$ เป็นเงื่อนไขของ $AM.-GM.$
แต่ในที่นี้ คุณกิตติใช้ $a_1=(xy)^2\geqslant 0$ และ $a_2=(yz)^2\geqslant 0$ ก็ถูกแล้วนี่ครับ
__________________
||!<<<<iNesZaii>>>>!||
ตอบพร้อมอ้างอิงข้อความนี้
  #41  
Old 30 เมษายน 2010, 18:01
กิตติ's Avatar
กิตติ กิตติ ไม่อยู่ในระบบ
กระบี่ธรรมชาติ
 
วันที่สมัครสมาชิก: 08 พฤศจิกายน 2009
ข้อความ: 2,723
กิตติ is on a distinguished road
Default

ใช่จริงๆด้วย เพราะกำลังสองกำลังสี่ยังไงก็เป็นจำนวนเต็มบวกที่มากกว่าหรือเท่ากับศูนย์
ดังนั้นวิธีแรกที่ผมใช้การพิสูจน์ที่อยู่ในเทอมของ$x,y,z$โดยตรงเลยจึงใช้ไม่ได้
แต่สามารถนำ$x^2,x^4,y^2,y^4$มาใช้แบบ$AM-GM$ได้
ขอบคุณทั้งคุณScylla_ShadowกับคุณNe[S]zA ทั้งสองท่านเลยครับที่ช่วยชี้ทางให้
__________________
"ถ้าเราล้มบ่อยๆ ในที่สุดเราจะรู้ว่าถ้าจะล้ม ล้มท่าไหนจะเจ็บน้อยที่สุด และรู้อีกว่าต่อไปทำยังไงจะไม่ให้ล้มอีก
ดังนั้นจงอย่ากลัวที่จะล้ม
"...อาจารย์อำนวย ขนันไทย
ครั้งแรกในชีวิตที่สอบคณิตสมาคมคณิตศาสตร์เมื่อปี2533...ผมได้แค่24คะแนน(จากร้อยคะแนน)

30 เมษายน 2010 18:01 : ข้อความนี้ถูกแก้ไขแล้ว 1 ครั้ง, ครั้งล่าสุดโดยคุณ กิตติ
ตอบพร้อมอ้างอิงข้อความนี้
ตั้งหัวข้อใหม่ Reply



กฎการส่งข้อความ
คุณ ไม่สามารถ ตั้งหัวข้อใหม่ได้
คุณ ไม่สามารถ ตอบหัวข้อได้
คุณ ไม่สามารถ แนบไฟล์และเอกสารได้
คุณ ไม่สามารถ แก้ไขข้อความของคุณเองได้

vB code is On
Smilies are On
[IMG] code is On
HTML code is Off
ทางลัดสู่ห้อง


เวลาที่แสดงทั้งหมด เป็นเวลาที่ประเทศไทย (GMT +7) ขณะนี้เป็นเวลา 23:28


Powered by vBulletin® Copyright ©2000 - 2024, Jelsoft Enterprises Ltd.
Modified by Jetsada Karnpracha